Last visit was: 19 Nov 2025, 01:04 It is currently 19 Nov 2025, 01:04
Close
GMAT Club Daily Prep
Thank you for using the timer - this advanced tool can estimate your performance and suggest more practice questions. We have subscribed you to Daily Prep Questions via email.

Customized
for You

we will pick new questions that match your level based on your Timer History

Track
Your Progress

every week, we’ll send you an estimated GMAT score based on your performance

Practice
Pays

we will pick new questions that match your level based on your Timer History
Not interested in getting valuable practice questions and articles delivered to your email? No problem, unsubscribe here.
Close
Request Expert Reply
Confirm Cancel
655-705 Level|   Weaken|                        
User avatar
AryamaDuttaSaikia
User avatar
Jamboree GMAT Instructor
Joined: 15 Jul 2015
Last visit: 06 Dec 2019
Posts: 252
Own Kudos:
693
 [2]
Given Kudos: 1
Status:GMAT Expert
Affiliations: Jamboree Education Pvt Ltd
Location: India
Posts: 252
Kudos: 693
 [2]
1
Kudos
Add Kudos
1
Bookmarks
Bookmark this Post
avatar
OptimusPrepJanielle
Joined: 06 Nov 2014
Last visit: 08 Sep 2017
Posts: 1,779
Own Kudos:
1,483
 [1]
Given Kudos: 23
Expert
Expert reply
Posts: 1,779
Kudos: 1,483
 [1]
Kudos
Add Kudos
1
Bookmarks
Bookmark this Post
User avatar
TeamGMATIFY
Joined: 20 Aug 2015
Last visit: 31 Oct 2016
Posts: 339
Own Kudos:
1,504
 [2]
Given Kudos: 10
Location: India
GMAT 1: 760 Q50 V44
Expert
Expert reply
GMAT 1: 760 Q50 V44
Posts: 339
Kudos: 1,504
 [2]
Kudos
Add Kudos
2
Bookmarks
Bookmark this Post
User avatar
zoezhuyan
Joined: 17 Sep 2016
Last visit: 11 Nov 2024
Posts: 418
Own Kudos:
Given Kudos: 147
Posts: 418
Kudos: 94
Kudos
Add Kudos
Bookmarks
Bookmark this Post
GMATNinja

Quote:
(B) Most of the people who bought Dietz's canned tuna for the first time as a result of the campaign were already loyal customers of other Dietz products.
A counterargument to the author's argument might be that the ad campaign expanded Dietz's customer base and, thus, that the initial cost of the ad campaign might be outweighed by increased profits in years to come. However, choice (B) eliminates this possible counterargument and thus strengthens the author's argument. Since we are looking for a weakening statement, choice (B) can be eliminated.


Dear GMATNinja and other experts, gmat1393, GMATNinjaTwo, nightblade354

although I read the whole tread, I am not convinced why B is incorrect.
I have a different interpretation of B, which still makes me think B is the correct answer.

B says ad champing attracts new customers who are already loyal customers of other Dietz produces. I thinks B does imply expansion of the customer base, although the number of the customer may be the same. no matter whether the number of customers increases or keeps same,B does expand the customer base. what impact the profit does not depend on how many people buy, but how much they pay.
in other words, a group of customers generally buy other Dietz products, after the ad champing, the same group customers will buy canned tuna that they did not buy in the past and continue to buy other Dietz products.

although as stimulus says ad champing attracted new customers, I think B does not contrast what stimulus says, new customers are those who did not not buy canned tuna but will begin to do so.

please help...
User avatar
GMATNinja
User avatar
GMAT Club Verbal Expert
Joined: 13 Aug 2009
Last visit: 18 Nov 2025
Posts: 7,445
Own Kudos:
69,781
 [2]
Given Kudos: 2,060
Status: GMAT/GRE/LSAT tutors
Location: United States (CO)
GMAT 1: 780 Q51 V46
GMAT 2: 800 Q51 V51
GRE 1: Q170 V170
GRE 2: Q170 V170
Products:
Expert
Expert reply
GMAT 2: 800 Q51 V51
GRE 1: Q170 V170
GRE 2: Q170 V170
Posts: 7,445
Kudos: 69,781
 [2]
2
Kudos
Add Kudos
Bookmarks
Bookmark this Post
zoezhuyan
GMATNinja

Quote:
(B) Most of the people who bought Dietz's canned tuna for the first time as a result of the campaign were already loyal customers of other Dietz products.
A counterargument to the author's argument might be that the ad campaign expanded Dietz's customer base and, thus, that the initial cost of the ad campaign might be outweighed by increased profits in years to come. However, choice (B) eliminates this possible counterargument and thus strengthens the author's argument. Since we are looking for a weakening statement, choice (B) can be eliminated.


Dear GMATNinja and other experts, gmat1393, GMATNinjaTwo, nightblade354

although I read the whole tread, I am not convinced why B is incorrect.
I have a different interpretation of B, which still makes me think B is the correct answer.

B says ad champing attracts new customers who are already loyal customers of other Dietz produces. I thinks B does imply expansion of the customer base, although the number of the customer may be the same. no matter whether the number of customers increases or keeps same,B does expand the customer base. what impact the profit does not depend on how many people buy, but how much they pay.
in other words, a group of customers generally buy other Dietz products, after the ad champing, the same group customers will buy canned tuna that they did not buy in the past and continue to buy other Dietz products.

although as stimulus says ad champing attracted new customers, I think B does not contrast what stimulus says, new customers are those who did not not buy canned tuna but will begin to do so.

please help...
Remember that our overall goal here is to weaken the argument that the canned tuna advertising campaign "did nothing to further Dietz's economic interests."

In line with your analysis, (B) tells us that most of the new tuna-buyers were already loyal customers of other Dietz products! So yes, it is completely possible that these same customers now spend more money on Dietz products, because now they also buy canned tuna.

But how does this actually impact the author's argument? The author already acknowledges that Dietz made additional sales of tuna after the ad campaign. Unfortunately, those additional sales did not further Dietz' economic interest, because the cost of the campaign outweighed the sales. (B) does not provide an explanation to show that the ad campaign actually benefited Dietz despite its high cost.

Now, consider the scenario in which the new customers attracted by the ad campaign were not already loyal Dietz customers. Then, perhaps these people would not only start buying tuna, but would also start to buy additional Dietz products. This could provide additional revenue, which could potentially mean that the tuna ad campaign was in Dietz' economic interest despite the costs outweighing the new sales in canned tuna. Answer choice (B) eliminates this possibility -- most new tuna customers already bought other Dietz products, so the ad campaign would not result in this additional revenue.

Answer choice (B) doesn't weaken the argument that the ad campaign "did nothing to further Dietz's economic interests," so (B) is out.

I hope that helps!
avatar
aritrar4
avatar
Current Student
Joined: 12 Jun 2020
Last visit: 06 Sep 2024
Posts: 103
Own Kudos:
Given Kudos: 147
Location: India
GMAT 1: 680 Q47 V35
GMAT 2: 690 Q49 V34
GMAT 3: 710 Q50 V35
GPA: 3.73
Products:
Kudos
Add Kudos
Bookmarks
Bookmark this Post
GMATNinja
This passage has a readily identifiable conclusion, so let's start with that. We are told that, "a year ago, Dietz Foods launched a yearlong advertising campaign for its canned tuna." The conclusion of the passage is that "the campaign did nothing to further Dietz's economic interests."

How does the author arrive at that conclusion?

  • Dietz sold 12 million cans of tuna last year (while the ad campaign was running).
  • Dietz sold only 10 million cans of tuna the previous year (before the ad campaign was started).
  • We are specifically told that this increase in sales was "directly attributable to new customers brought in by the campaign." Because this information is given, we don't have to worry about whether any assumptions are required to arrive at this intermediate conclusion.
  • However, profits from these additional sales were substantially less the cost of the ad campaign.
  • Since the costs substantially exceeded the profits, the author concludes that the ad campaign did nothing to further the company's economic interests.

Now we need to find an answer choice that, if true, most seriously weakens this argument:

Quote:
(A) Sales of canned tuna account for a relatively small percentage of Dietz Foods' profits.
Last year, the cost of the ad campaign exceeded the additional profits created by the ad campaign. Based on those facts, the author concludes that the campaign did not further the company's economic interests. Regardless of the percentage of total profits accounted for by sales of canned tuna, if costs exceeded profits, according to the author, the campaign did not further the company's economic interests. Choice (A) does not interfere with this logic and can be eliminated.

Quote:
(B) Most of the people who bought Dietz's canned tuna for the first time as a result of the campaign were already loyal customers of other Dietz products.
A counterargument to the author's argument might be that the ad campaign expanded Dietz's customer base and, thus, that the initial cost of the ad campaign might be outweighed by increased profits in years to come. However, choice (B) eliminates this possible counterargument and thus strengthens the author's argument. Since we are looking for a weakening statement, choice (B) can be eliminated.

Quote:
(C) A less expensive advertising campaign would have brought in significantly fewer new customers for Dietz's canned tuna than did the campaign Dietz Foods launched last year.
Choice (C) does not change the fact that the cost of last year's ad campaign did not exceed the additional profits created by the ad campaign last year. Choice (C) does not impact the author's reasoning and, thus, can be eliminated.

Quote:
(D) Dietz made money on sales of canned tuna last year.
We are told that Dietz profited from the additional sales ("Profits from the additional sales..."). Choice (D) does not give us any new information and does not change the fact that the campaign's costs exceeded the additional profits last year. Eliminate (D).

Quote:
(E) In each of the past five years, there was a steep, industry-wide decline in sales of canned tuna.
According to the author's argument, the company would have been better off economically if it had not run the ad campaign. The ad campaign seemingly increased costs more than it increased profits. This analysis rests on the assumption that profits would have remained the same (compared to the previous year) if the ad campaign had not been run.

But what if profits would have decreased if the ad campaign had not been run? In that case, we would have to compare the cost of the ad campaign not just to the increase in profits but to the sum of the increase in profits and the potential profit loss. Perhaps the cost of the ad campaign exceeded this sum, in which case the author's logic would fail. Choice (E) allows for this possibility by suggesting that Dietz's sales of canned tuna would have been much less than 10 million if it weren't for the ad campaign.

Choice (E) most seriously weakens the author's argument and, thus, is the best answer.

GMATNinja - Could you please help clarify a doubt I'm having regarding option C?

In the above explanation you stated that the cost of last year's campaign did not exceed the additional profits created by the ad campaign. I think it would be "did exceed", however I have another doubt regarding this one. If a cheaper ad would have meant fewer customers, it would have meant lesser sales too, making choice C the better option in my mind. Can this assumption be negated by thinking that "fewer customers does not necessarily mean lesser sales"? In that case choice C can be eliminated. Thanks for your help !
User avatar
peanuts
User avatar
Current Student
Joined: 17 May 2020
Last visit: 29 May 2021
Posts: 61
Own Kudos:
57
 [1]
Given Kudos: 34
Location: Viet Nam
GMAT 1: 680 Q49 V34
GMAT 2: 720 Q50 V38
Kudos
Add Kudos
1
Bookmarks
Bookmark this Post
A year ago, Dietz Foods launched a yearlong advertising campaign for its canned tuna. Last year Dietz sold 12 million cans of tuna compared to the 10 million sold during the previous year, an increase directly attributable to new customers brought in by the campaign. Profits from the additional sales, however, were substantially less than the cost of the advertising campaign. Clearly, therefore, the campaign did nothing to further Dietz's economic interests.

Premise 1: Last year, Dietz sold 12m cans of tuna compared to 10m the previous year
Premise 2: profit from the sales was less than cost of advertising

Conclusion: Advertising does nothing in terms of economic interest for Dietz

Which of the following, if true, most seriously weakens the argument?

To weaken this argument, we should find the answer that prove that even with the profit smaller than the cost, the campaign still bring economic benefit to Dietz

(A) Sales of canned tuna account for a relatively small percentage of Dietz Foods' profits. So what? We don't care how much the canned tuna accounted for in the total profit of Dietz

(B) Most of the people who bought Dietz's canned tuna for the first time as a result of the campaign were already loyal customers of other Dietz products. It means that even with the ad, new customers are just loyal customer already, so the ad did almost nothing to attract new customer

(C) A less expensive advertising campaign would have brought in significantly fewer new customers for Dietz's canned tuna than did the campaign Dietz Foods launched last year. So the ad effect will be down, just because the investment in the ad is not enough

(D) Dietz made money on sales of canned tuna last year. So what? We care about effect of the ad

(E) In each of the past five years, there was a steep, industry-wide decline in sales of canned tuna. Oh yes. Even in the general trend of decline in sales of canned tuna, the ad helped to increase customer of Dietz to its tuna. It clearly furthers the economic interest of the company
User avatar
GMATGuruNY
Joined: 04 Aug 2010
Last visit: 18 Nov 2025
Posts: 1,344
Own Kudos:
3,796
 [1]
Given Kudos: 9
Schools:Dartmouth College
Expert
Expert reply
Posts: 1,344
Kudos: 3,796
 [1]
1
Kudos
Add Kudos
Bookmarks
Bookmark this Post
ExecMBA2010
A year ago, Dietz Foods launched a yearlong advertising campaign for its canned tuna. Last year Dietz sold 12 million cans of tuna compared to the 10 million sold during the previous year, an increase directly attributable to new customers brought in by the campaign. Profits from the additional sales, however, were substantially less than the cost of the advertising campaign. Clearly, therefore, the campaign did nothing to further Dietz's economic interests.

Which of the following, if true, most seriously weakens the argument?


(A) Sales of canned tuna account for a relatively small percentage of Dietz Foods' profits.

(B) Most of the people who bought Dietz's canned tuna for the first time as a result of the campaign were already loyal customers of other Dietz products.

(C) A less expensive advertising campaign would have brought in significantly fewer new customers for Dietz's canned tuna than did the campaign Dietz Foods launched last year.

(D) Dietz made money on sales of canned tuna last year.

(E) In each of the past five years, there was a steep, industry-wide decline in sales of canned tuna.

Premise:
Profits from the additional sales, however, were substantially less than the cost of the advertising campaign.
Conclusion:
The campaign did nothing to further Dietz's economic interests.

To weaken the conclusion, the correct answer must show that the campaign DID somehow further Dietz's economic interests.

E: In each of the past five years, there was a steep, industry-wide decline in sales of canned tuna.
While the rest of the industry suffered a steep DECLINE in sales, the campaign enabled Dietz to INCREASE its sales.
Implication:
Because of the campaign, Dietz avoided the steep decline in sales suffered by the rest of the industry -- a clear furtherance of the company's economic interests.

avatar
KumarMohan
Joined: 28 Apr 2019
Last visit: 31 Dec 2024
Posts: 9
Own Kudos:
3
 [1]
Given Kudos: 7
Posts: 9
Kudos: 3
 [1]
1
Kudos
Add Kudos
Bookmarks
Bookmark this Post
I gone though all the replies mentioned in the quetions and no one seems to be talking about B.

In option B "Most of the people who bought Dietz's canned tuna for the first time as a result of the campaign were already loyal customers of other Dietz products."

From this we can say that since these are loyal customer who previously did not buy canned tuna (reason for which may be anything) but now this year because of campaign they have brought it so going forward they are expected to buy canned tuna even when there is no compaign because they are loyal to the company.

Can someone pls help me understand flow in this reasoning..

Thanks in advance for your help
User avatar
GMATGuruNY
Joined: 04 Aug 2010
Last visit: 18 Nov 2025
Posts: 1,344
Own Kudos:
Given Kudos: 9
Schools:Dartmouth College
Expert
Expert reply
Posts: 1,344
Kudos: 3,796
Kudos
Add Kudos
Bookmarks
Bookmark this Post
KumarMohan
I gone though all the replies mentioned in the quetions and no one seems to be talking about B.

In option B "Most of the people who bought Dietz's canned tuna for the first time as a result of the campaign were already loyal customers of other Dietz products."

From this we can say that since these are loyal customer who previously did not buy canned tuna (reason for which may be anything) but now this year because of campaign they have brought it so going forward they are expected to buy canned tuna even when there is no compaign because they are loyal to the company.

Can someone pls help me understand flow in this reasoning..

Thanks in advance for your help

Since the correct answer must WEAKEN the conclusion, eliminate any answer choice that could serve to strengthen it.
B: Most of the people who bought Dietz's canned tuna for the first time as a result of the campaign were already loyal customers of other Dietz products.
Here, most of the first-time buyers of tuna were already loyal customers of Dietz Foods, implying that the company's customer base did not significantly increase as a result of the ad campaign -- information that could serve to STRENGTHEN the conclusion that the campaign did not further the company's economic interests.
Eliminate B.
User avatar
GMATNinja
User avatar
GMAT Club Verbal Expert
Joined: 13 Aug 2009
Last visit: 18 Nov 2025
Posts: 7,445
Own Kudos:
69,781
 [1]
Given Kudos: 2,060
Status: GMAT/GRE/LSAT tutors
Location: United States (CO)
GMAT 1: 780 Q51 V46
GMAT 2: 800 Q51 V51
GRE 1: Q170 V170
GRE 2: Q170 V170
Products:
Expert
Expert reply
GMAT 2: 800 Q51 V51
GRE 1: Q170 V170
GRE 2: Q170 V170
Posts: 7,445
Kudos: 69,781
 [1]
1
Kudos
Add Kudos
Bookmarks
Bookmark this Post
aritrar4
GMATNinja
This passage has a readily identifiable conclusion, so let's start with that. We are told that, "a year ago, Dietz Foods launched a yearlong advertising campaign for its canned tuna." The conclusion of the passage is that "the campaign did nothing to further Dietz's economic interests."

How does the author arrive at that conclusion?

  • Dietz sold 12 million cans of tuna last year (while the ad campaign was running).
  • Dietz sold only 10 million cans of tuna the previous year (before the ad campaign was started).
  • We are specifically told that this increase in sales was "directly attributable to new customers brought in by the campaign." Because this information is given, we don't have to worry about whether any assumptions are required to arrive at this intermediate conclusion.
  • However, profits from these additional sales were substantially less the cost of the ad campaign.
  • Since the costs substantially exceeded the profits, the author concludes that the ad campaign did nothing to further the company's economic interests.

Now we need to find an answer choice that, if true, most seriously weakens this argument:

Quote:
(A) Sales of canned tuna account for a relatively small percentage of Dietz Foods' profits.
Last year, the cost of the ad campaign exceeded the additional profits created by the ad campaign. Based on those facts, the author concludes that the campaign did not further the company's economic interests. Regardless of the percentage of total profits accounted for by sales of canned tuna, if costs exceeded profits, according to the author, the campaign did not further the company's economic interests. Choice (A) does not interfere with this logic and can be eliminated.

Quote:
(B) Most of the people who bought Dietz's canned tuna for the first time as a result of the campaign were already loyal customers of other Dietz products.
A counterargument to the author's argument might be that the ad campaign expanded Dietz's customer base and, thus, that the initial cost of the ad campaign might be outweighed by increased profits in years to come. However, choice (B) eliminates this possible counterargument and thus strengthens the author's argument. Since we are looking for a weakening statement, choice (B) can be eliminated.

Quote:
(C) A less expensive advertising campaign would have brought in significantly fewer new customers for Dietz's canned tuna than did the campaign Dietz Foods launched last year.
Choice (C) does not change the fact that the cost of last year's ad campaign did not exceed the additional profits created by the ad campaign last year. Choice (C) does not impact the author's reasoning and, thus, can be eliminated.

Quote:
(D) Dietz made money on sales of canned tuna last year.
We are told that Dietz profited from the additional sales ("Profits from the additional sales..."). Choice (D) does not give us any new information and does not change the fact that the campaign's costs exceeded the additional profits last year. Eliminate (D).

Quote:
(E) In each of the past five years, there was a steep, industry-wide decline in sales of canned tuna.
According to the author's argument, the company would have been better off economically if it had not run the ad campaign. The ad campaign seemingly increased costs more than it increased profits. This analysis rests on the assumption that profits would have remained the same (compared to the previous year) if the ad campaign had not been run.

But what if profits would have decreased if the ad campaign had not been run? In that case, we would have to compare the cost of the ad campaign not just to the increase in profits but to the sum of the increase in profits and the potential profit loss. Perhaps the cost of the ad campaign exceeded this sum, in which case the author's logic would fail. Choice (E) allows for this possibility by suggesting that Dietz's sales of canned tuna would have been much less than 10 million if it weren't for the ad campaign.

Choice (E) most seriously weakens the author's argument and, thus, is the best answer.

GMATNinja - Could you please help clarify a doubt I'm having regarding option C?

In the above explanation you stated that the cost of last year's campaign did not exceed the additional profits created by the ad campaign. I think it would be "did exceed", however I have another doubt regarding this one. If a cheaper ad would have meant fewer customers, it would have meant lesser sales too, making choice C the better option in my mind. Can this assumption be negated by thinking that "fewer customers does not necessarily mean lesser sales"? In that case choice C can be eliminated. Thanks for your help !
You’re right that our previous post should say that “the cost of last year’s ad campaign exceeded the additional profits created by the ad campaign last year” rather than “did not exceed.”

Regarding (C), the author concludes that the ad campaign did not further the company’s economic interests because costs exceeded profits. If a cheaper ad campaign would lead to fewer sales, then a cheaper ad campaign may not be preferable. But this does nothing to prove that the actual ad campaign was economically beneficial. The ad campaign’s costs still exceeded the profits from additional sales.

In other words, simply pointing out that an alternative course of action (a less expensive ad campaign) produces a less desirable outcome (fewer new customers) does not necessarily mean that the original course of action (the actual ad campaign) produces a desirable outcome (furthering Dietz’s economic interests). For that reason, (C) does not weaken the argument, and we can eliminate it.

I hope that helps!
avatar
bahruz1992
Joined: 30 Mar 2020
Last visit: 25 Mar 2022
Posts: 17
Own Kudos:
Given Kudos: 34
Posts: 17
Kudos: 11
Kudos
Add Kudos
Bookmarks
Bookmark this Post
GMATNinja
This passage has a readily identifiable conclusion, so let's start with that. We are told that, "a year ago, Dietz Foods launched a yearlong advertising campaign for its canned tuna." The conclusion of the passage is that "the campaign did nothing to further Dietz's economic interests."

How does the author arrive at that conclusion?

  • Dietz sold 12 million cans of tuna last year (while the ad campaign was running).
  • Dietz sold only 10 million cans of tuna the previous year (before the ad campaign was started).
  • We are specifically told that this increase in sales was "directly attributable to new customers brought in by the campaign." Because this information is given, we don't have to worry about whether any assumptions are required to arrive at this intermediate conclusion.
  • However, profits from these additional sales were substantially less the cost of the ad campaign.
  • Since the costs substantially exceeded the profits, the author concludes that the ad campaign did nothing to further the company's economic interests.

Now we need to find an answer choice that, if true, most seriously weakens this argument:

Quote:
(A) Sales of canned tuna account for a relatively small percentage of Dietz Foods' profits.
Last year, the cost of the ad campaign exceeded the additional profits created by the ad campaign. Based on those facts, the author concludes that the campaign did not further the company's economic interests. Regardless of the percentage of total profits accounted for by sales of canned tuna, if costs exceeded profits, according to the author, the campaign did not further the company's economic interests. Choice (A) does not interfere with this logic and can be eliminated.

Quote:
(B) Most of the people who bought Dietz's canned tuna for the first time as a result of the campaign were already loyal customers of other Dietz products.
A counterargument to the author's argument might be that the ad campaign expanded Dietz's customer base and, thus, that the initial cost of the ad campaign might be outweighed by increased profits in years to come. However, choice (B) eliminates this possible counterargument and thus strengthens the author's argument. Since we are looking for a weakening statement, choice (B) can be eliminated.

Quote:
(C) A less expensive advertising campaign would have brought in significantly fewer new customers for Dietz's canned tuna than did the campaign Dietz Foods launched last year.
Choice (C) does not change the fact that the cost of last year's ad campaign exceeded the additional profits created by the ad campaign last year. Choice (C) does not impact the author's reasoning and, thus, can be eliminated.

Quote:
(D) Dietz made money on sales of canned tuna last year.
We are told that Dietz profited from the additional sales ("Profits from the additional sales..."). Choice (D) does not give us any new information and does not change the fact that the campaign's costs exceeded the additional profits last year. Eliminate (D).

Quote:
(E) In each of the past five years, there was a steep, industry-wide decline in sales of canned tuna.
According to the author's argument, the company would have been better off economically if it had not run the ad campaign. The ad campaign seemingly increased costs more than it increased profits. This analysis rests on the assumption that profits would have remained the same (compared to the previous year) if the ad campaign had not been run.

But what if profits would have decreased if the ad campaign had not been run? In that case, we would have to compare the cost of the ad campaign not just to the increase in profits but to the sum of the increase in profits and the potential profit loss. Perhaps the cost of the ad campaign exceeded this sum, in which case the author's logic would fail. Choice (E) allows for this possibility by suggesting that Dietz's sales of canned tuna would have been much less than 10 million if it weren't for the ad campaign.

Choice (E) most seriously weakens the author's argument and, thus, is the best answer.

Thank you very much for explanation. Can we interpret answer (B) in this way: We know that Dietz has some loyal customer base that until now purchased some different products of the Company again and again. We know that those customers are loyal and, as answer (B) implies they purchased the canned tuna first time. As those customer base is loyal, it would be correct to assume that those loyal customers will buy canned tuna in upcoming years. As a results, in upcoming years, ad campaigns will have a positive impact on the Dietz's sales. SO, answer (B) weakens the conclusion.
User avatar
GMATNinja
User avatar
GMAT Club Verbal Expert
Joined: 13 Aug 2009
Last visit: 18 Nov 2025
Posts: 7,445
Own Kudos:
69,781
 [1]
Given Kudos: 2,060
Status: GMAT/GRE/LSAT tutors
Location: United States (CO)
GMAT 1: 780 Q51 V46
GMAT 2: 800 Q51 V51
GRE 1: Q170 V170
GRE 2: Q170 V170
Products:
Expert
Expert reply
GMAT 2: 800 Q51 V51
GRE 1: Q170 V170
GRE 2: Q170 V170
Posts: 7,445
Kudos: 69,781
 [1]
1
Kudos
Add Kudos
Bookmarks
Bookmark this Post
bahruz1992

Thank you very much for explanation. Can we interpret answer (B) in this way: We know that Dietz has some loyal customer base that until now purchased some different products of the Company again and again. We know that those customers are loyal and, as answer (B) implies they purchased the canned tuna first time. As those customer base is loyal, it would be correct to assume that those loyal customers will buy canned tuna in upcoming years. As a results, in upcoming years, ad campaigns will have a positive impact on the Dietz's sales. SO, answer (B) weakens the conclusion.
It’s probably a bit of a stretch to conclude that, because Dietz’s customers were loyal purchasers of other products, they will be loyal purchasers of the canned tuna as well. But even if we could find a situation in which (B) would weaken the argument, it still would not be the best answer choice.

As we laid out in our explanation above, it’s definitely possible that (B) strengthens the argument. And when an answer choice could strengthen or could weaken an argument depending on the circumstances, it’s definitely not a serious weakener. So we can eliminate it.

I hope that helps!
User avatar
GMATE1
Joined: 22 Nov 2020
Last visit: 31 Dec 2021
Posts: 61
Own Kudos:
Given Kudos: 163
Posts: 61
Kudos: 24
Kudos
Add Kudos
Bookmarks
Bookmark this Post
GMATNinja
bahruz1992

Thank you very much for explanation. Can we interpret answer (B) in this way: We know that Dietz has some loyal customer base that until now purchased some different products of the Company again and again. We know that those customers are loyal and, as answer (B) implies they purchased the canned tuna first time. As those customer base is loyal, it would be correct to assume that those loyal customers will buy canned tuna in upcoming years. As a results, in upcoming years, ad campaigns will have a positive impact on the Dietz's sales. SO, answer (B) weakens the conclusion.
It’s probably a bit of a stretch to conclude that, because Dietz’s customers were loyal purchasers of other products, they will be loyal purchasers of the canned tuna as well. But even if we could find a situation in which (B) would weaken the argument, it still would not be the best answer choice.

As we laid out in our explanation above, it’s definitely possible that (B) strengthens the argument. And when an answer choice could strengthen or could weaken an argument depending on the circumstances, it’s definitely not a serious weakener. So we can eliminate it.

I hope that helps!

I had the same thought. From B we know that the people who bought Dietz's canned tuna for the first time as a result of the campaign were already loyal customers of other Dietz products. Due to the fact that these customers are loyal to other Dietz products it seems not far fetched that these customers will also be likely to Diet's canned tunna. This would mean that the campaign expenses could be attributed not only to the 2 million additional tuna cans sold but also to many more millions of tuna cans sold in the following years by these concumers.

Therefore I believe that B weakens the argument most, because it seems that the campaign attracted loyal customers which are likely to become repeat customers. And while on a one-year perspective there might not have been an economic gains over multiple years the repeat purchases should outweight the cost of the campaign.

To me this question seems flawed, as it fails to specify the time frame, which we are supposed to use. If we are only looking at a narrow time frame E would be the better choice but on a more long term view B seems better, as repeat customers of a product are a lot more important than customers, who only purchase an item once.

GMATNinja
But what if profits would have decreased if the ad campaign had not been run? In that case, we would have to compare the cost of the ad campaign not just to the increase in profits but to the sum of the increase in profits and the potential profit loss. Perhaps the cost of the ad campaign exceeded this sum, in which case the author's logic would fail. Choice (E) allows for this possibility by suggesting that Dietz's sales of canned tuna would have been much less than 10 million if it weren't for the ad campaign.

What bothers me about this explanations is the following: The entire argument relies on the assumption that because there was an industry-wide decline in sales of canned tuna it was likely that Dietz's sales would have decreased this year without the campaign. This argument is flawed for 2 reasons: 1 past behaviour does not predict future behaviour (at least it does not have to). It is possible that sales will decrease again this year for the same reason they did in the past, but without knowing the reason for the decline we will not know whether they will decrease again. 2. Only because the market for canned tuna decreased during the past 5 years does not mean that Dietz's sales have decreased or will decrease. For example cars sales declined in 2020 but Teslas car sales increased. (There are countless other examples inwhich there is a decrease in sales industry wide but individual companies grow their sales.) The reason for this is that industry wide sales are the total sales of an industry but we are concerned with the toal sales not of an industry but a specific company.
User avatar
Tanchat
Joined: 31 Jan 2020
Last visit: 20 Jun 2023
Posts: 222
Own Kudos:
Given Kudos: 139
Posts: 222
Kudos: 20
Kudos
Add Kudos
Bookmarks
Bookmark this Post
GMATNinja
bahruz1992

Thank you very much for explanation. Can we interpret answer (B) in this way: We know that Dietz has some loyal customer base that until now purchased some different products of the Company again and again. We know that those customers are loyal and, as answer (B) implies they purchased the canned tuna first time. As those customer base is loyal, it would be correct to assume that those loyal customers will buy canned tuna in upcoming years. As a results, in upcoming years, ad campaigns will have a positive impact on the Dietz's sales. SO, answer (B) weakens the conclusion.
It’s probably a bit of a stretch to conclude that, because Dietz’s customers were loyal purchasers of other products, they will be loyal purchasers of the canned tuna as well. But even if we could find a situation in which (B) would weaken the argument, it still would not be the best answer choice.

As we laid out in our explanation above, it’s definitely possible that (B) strengthens the argument. And when an answer choice could strengthen or could weaken an argument depending on the circumstances, it’s definitely not a serious weakener. So we can eliminate it.

I hope that helps!

GMATNinja
I have 2 questions

1) "Clearly, therefore, the campaign did nothing to further Dietz's economic interests." << What does this mean? Can we interpret that campaign could make "positive or negative" impact on economic interests? or does it have to be positive impact or it can be either.

2) I think (E) could strength or could weaken an argument too.
- > Although industry wide decline in sales of canned tuna, Dietz may make profits from doing nothing more than profits from doing campaign.
or
- > If Dietz didn't create the campaign, Dietz may lost more profits.
User avatar
GMATNinja
User avatar
GMAT Club Verbal Expert
Joined: 13 Aug 2009
Last visit: 18 Nov 2025
Posts: 7,445
Own Kudos:
69,781
 [1]
Given Kudos: 2,060
Status: GMAT/GRE/LSAT tutors
Location: United States (CO)
GMAT 1: 780 Q51 V46
GMAT 2: 800 Q51 V51
GRE 1: Q170 V170
GRE 2: Q170 V170
Products:
Expert
Expert reply
GMAT 2: 800 Q51 V51
GRE 1: Q170 V170
GRE 2: Q170 V170
Posts: 7,445
Kudos: 69,781
 [1]
1
Kudos
Add Kudos
Bookmarks
Bookmark this Post
Tanchat
GMATNinja
bahruz1992

Thank you very much for explanation. Can we interpret answer (B) in this way: We know that Dietz has some loyal customer base that until now purchased some different products of the Company again and again. We know that those customers are loyal and, as answer (B) implies they purchased the canned tuna first time. As those customer base is loyal, it would be correct to assume that those loyal customers will buy canned tuna in upcoming years. As a results, in upcoming years, ad campaigns will have a positive impact on the Dietz's sales. SO, answer (B) weakens the conclusion.
It’s probably a bit of a stretch to conclude that, because Dietz’s customers were loyal purchasers of other products, they will be loyal purchasers of the canned tuna as well. But even if we could find a situation in which (B) would weaken the argument, it still would not be the best answer choice.

As we laid out in our explanation above, it’s definitely possible that (B) strengthens the argument. And when an answer choice could strengthen or could weaken an argument depending on the circumstances, it’s definitely not a serious weakener. So we can eliminate it.

I hope that helps!

GMATNinja
I have 2 questions

1) "Clearly, therefore, the campaign did nothing to further Dietz's economic interests." << What does this mean? Can we interpret that campaign could make "positive or negative" impact on economic interests? or does it have to be positive impact or it can be either.

2) I think (E) could strength or could weaken an argument too.
- > Although industry wide decline in sales of canned tuna, Dietz may make profits from doing nothing more than profits from doing campaign.
or
- > If Dietz didn't create the campaign, Dietz may lost more profits.
To answer your first question: "furthering" one's economic interests means having a positive impact on those interests. The author argues that the tuna campaign did not further Dietz' economic interests -- in other words, the author thinks that the campaign did not have a positive impact on Dietz.

To answer your second question: I'm not sure I understand your reasoning, and it seems that both of those explanations suggest that the campaign had a positive impact on Dietz Food. For a full explanation of (E), check out this post and let us know if you have any further questions.
User avatar
Tanchat
Joined: 31 Jan 2020
Last visit: 20 Jun 2023
Posts: 222
Own Kudos:
Given Kudos: 139
Posts: 222
Kudos: 20
Kudos
Add Kudos
Bookmarks
Bookmark this Post
GMATNinja

I already read your full explanation of E, but I'm still doubtful

the reason I asked question 1 because I wanted to know that we are asked whether the campaign just makes an impact (either negative or positive) or (must) positive impact.
The 2nd question : what if Dietz didn't create campaign, Dietz might lost revenue and profit, but the profit from doing nothing might be higher than the profit from creating campaign. Therefore, the campaign made the negative impact on Dietz's econ. Therefore, this scenario strengthens the conclusion.
User avatar
GMATNinja
User avatar
GMAT Club Verbal Expert
Joined: 13 Aug 2009
Last visit: 18 Nov 2025
Posts: 7,445
Own Kudos:
69,781
 [1]
Given Kudos: 2,060
Status: GMAT/GRE/LSAT tutors
Location: United States (CO)
GMAT 1: 780 Q51 V46
GMAT 2: 800 Q51 V51
GRE 1: Q170 V170
GRE 2: Q170 V170
Products:
Expert
Expert reply
GMAT 2: 800 Q51 V51
GRE 1: Q170 V170
GRE 2: Q170 V170
Posts: 7,445
Kudos: 69,781
 [1]
1
Kudos
Add Kudos
Bookmarks
Bookmark this Post
Tanchat
GMATNinja

I already read your full explanation of E, but I'm still doubtful

the reason I asked question 1 because I wanted to know that we are asked whether the campaign just makes an impact (either negative or positive) or (must) positive impact.
The 2nd question : what if Dietz didn't create campaign, Dietz might lost revenue and profit, but the profit from doing nothing might be higher than the profit from creating campaign. Therefore, the campaign made the negative impact on Dietz's econ. Therefore, this scenario strengthens the conclusion.
The author concludes that "the campaign did nothing to further Dietz's economic interests." In other words, the author thinks the campaign failed to have a positive impact on "Dietz's economic interests." But how do they reach this conclusion?

Well, the passage points out that the "profits from additional sales" were "substantially less than the cost of the advertising campaign."

As you say, we don't know what would have happened if Dietz didn't create the campaign. Their profits might have been lower, or they might have been higher -- we simply don't know. But either way, the right answer should weaken the author's conclusion that the campaign "did nothing to further Dietz's economic interests."

Let's now consider (E):

Quote:
Which of the following, if true, most seriously weakens the argument?

(E) In each of the past five years, there was a steep, industry-wide decline in sales of canned tuna.
If there was an "industry-wide decline in sales of canned tuna," you'd expect Dietz to have sold fewer cans of tuna last year than the year before. But in fact, Dietz sold 2 million more cans of tuna last year. So despite the industry-wide decline, Dietz managed to sell even more tuna than usual. What could account for this?

Well, one explanation is that the advertising campaign caused Dietz to sell more tuna despite an industry-wide decline. And if that's the case, the advertising campaign did "further" its economic interests. So (E) weakens the argument that the campaign "did nothing to further Dietz's economic interests."

You're correct that we really don't know what would have happened if Dietz didn't create the campaign - their profits might have been smaller, or they might have been bigger. So (E) certainly doesn't PROVE that the campaign furthered Dietz's economic interests. However, it does provide evidence to suggest the campaign increased Dietz's sales of tuna. Because when they ran the campaign, they sold more tuna than usual despite an industry-wide decline.

So while (E) doesn't destroy the argument, it does weaken it, which makes it correct.

I hope that helps!
avatar
AcceleratorCC
Joined: 19 Mar 2022
Last visit: 13 Sep 2022
Posts: 10
Own Kudos:
Given Kudos: 11
Posts: 10
Kudos: 1
Kudos
Add Kudos
Bookmarks
Bookmark this Post
Quote:
I had the same thought. From B we know that the people who bought Dietz's canned tuna for the first time as a result of the campaign were already loyal customers of other Dietz products. Due to the fact that these customers are loyal to other Dietz products it seems not far fetched that these customers will also be likely to Diet's canned tunna. This would mean that the campaign expenses could be attributed not only to the 2 million additional tuna cans sold but also to many more millions of tuna cans sold in the following years by these customers.

Therefore I believe that B weakens the argument most, because it seems that the campaign attracted loyal customers which are likely to become repeat customers. And while on a one-year perspective there might not have been an economic gains over multiple years the repeat purchases should outweigh the cost of the campaign.

To me this question seems flawed, as it fails to specify the time frame, which we are supposed to use. If we are only looking at a narrow time frame E would be the better choice but on a more long term view B seems better, as repeat customers of a product are a lot more important than customers, who only purchase an item once.

I have the same question. Can any expert kindly help me?

"Most of the people who bought Dietz???s canned tuna for the first time as a result of the campaign were already loyal customers of other Dietz products".
For me, B weakens the argument because of the following reasoning.
1. The people were loyal customers of other Dietz products, but they didn't buy Dietz's canned tuna before, might because they didn't get a chance to know this product well in the past.
2. They bought Dietz's canned tuna for the first time after the campaign. Their purchases caused 2 million additional sales last year, though the profits were less than the costs of campaign.
3. They were loyal customers of Dietz who got to know and bought the canned tuna for the first time, so they are very likely to buy more Dietz canned tuna in the future and bring much more additional profits than the 2 million in the year of campaign.

So in the long run, the profits will outweigh the cost of the campaign. B weakens the argument.
User avatar
GMATNinja
User avatar
GMAT Club Verbal Expert
Joined: 13 Aug 2009
Last visit: 18 Nov 2025
Posts: 7,445
Own Kudos:
69,781
 [1]
Given Kudos: 2,060
Status: GMAT/GRE/LSAT tutors
Location: United States (CO)
GMAT 1: 780 Q51 V46
GMAT 2: 800 Q51 V51
GRE 1: Q170 V170
GRE 2: Q170 V170
Products:
Expert
Expert reply
GMAT 2: 800 Q51 V51
GRE 1: Q170 V170
GRE 2: Q170 V170
Posts: 7,445
Kudos: 69,781
 [1]
1
Kudos
Add Kudos
Bookmarks
Bookmark this Post
AcceleratorCC
Quote:
I had the same thought. From B we know that the people who bought Dietz's canned tuna for the first time as a result of the campaign were already loyal customers of other Dietz products. Due to the fact that these customers are loyal to other Dietz products it seems not far fetched that these customers will also be likely to Diet's canned tunna. This would mean that the campaign expenses could be attributed not only to the 2 million additional tuna cans sold but also to many more millions of tuna cans sold in the following years by these customers.

Therefore I believe that B weakens the argument most, because it seems that the campaign attracted loyal customers which are likely to become repeat customers. And while on a one-year perspective there might not have been an economic gains over multiple years the repeat purchases should outweigh the cost of the campaign.

To me this question seems flawed, as it fails to specify the time frame, which we are supposed to use. If we are only looking at a narrow time frame E would be the better choice but on a more long term view B seems better, as repeat customers of a product are a lot more important than customers, who only purchase an item once.

I have the same question. Can any expert kindly help me?

"Most of the people who bought Dietz???s canned tuna for the first time as a result of the campaign were already loyal customers of other Dietz products".
For me, B weakens the argument because of the following reasoning.
1. The people were loyal customers of other Dietz products, but they didn't buy Dietz's canned tuna before, might because they didn't get a chance to know this product well in the past.
2. They bought Dietz's canned tuna for the first time after the campaign. Their purchases caused 2 million additional sales last year, though the profits were less than the costs of campaign.
3. They were loyal customers of Dietz who got to know and bought the canned tuna for the first time, so they are very likely to buy more Dietz canned tuna in the future and bring much more additional profits than the 2 million in the year of campaign.

So in the long run, the profits will outweigh the cost of the campaign. B weakens the argument.
The passage tells us that Dietz lost money last year on the advertising campaign, despite attracting more customers. The fact that most of these customers were "already loyal customers of other Dietz products," as (B) tells us, doesn't directly impact this evidence. As a result, it doesn't weaken the conclusion that "the campaign did nothing to further Dietz's economic interests."

As you suggest, we could speculate about how the advertising campaign will impact Dietz's profits in the future. But to draw any definite conclusions about the future would requires big assumptions.

For example, the following are just a few of the assumptions we'd need to make for (B) to weaken the argument:

  • The new customers will continue to buy tuna at similar levels in future years, even without an advertising campaign
  • The amount of tuna sold in the future will outweigh the amount spent on the advertising campaign
  • Dietz will not encounter any competition that seriously reduces its sales of tuna in the future
  • Dietz continues to sell tuna in future years

In other words, for (B) to weaken the argument, we'd need to assume many things about the future. Since we can't make those assumptions, (B) on its own doesn't weaken the argument.

I hope that helps!
   1   2   3   
Moderators:
GMAT Club Verbal Expert
7445 posts
GMAT Club Verbal Expert
234 posts
188 posts